what is the answer for 14a³ - 22a we have to Factorise it​

Answers

Answer 1

Answer: 2a (7a² - 11).

Answer 2

Answer:

Step-by-step explanation:

Both numbers are even. You can take out a 2.

14/2 = 7

22/2 = 11

There is a limitation of one a on the 22. But you can take out 1 a

a^3/a = a^2

Combing you get

Answer: 2a(7a^2 - 11)

This is the reverse distributive property.


Related Questions

Lainey runs a string of lights from the ground straight up a door that is 2.5 m tall. Then he runs the rest of the string in a straight line to a point on the ground which is 6 m from the base of the door frame. There are 10 lights per meter of the string. How many total lights are on the string?

Answers

Answer:

150 because 2.5 × 6 = 15 × 10 = 150

write brackets () in this statement to make it correct 6 + 4 x 5 - 3 = 14

Answers

Answer:

6+4*(5-3)

Step-by-step explanation:

Because of PEMDAS l, we first solve in the parantheses. So 5-3 would be 2, then we multiply 2 by 4 which would be 8 obviously. Then we do the final step, add 6 and 8 which would be 14. Cheers!

Answer: 6 + 4 × (5-3)

Explanation:

6 + 4 × (5-3) = 14

6 + 4 × 2 = 14

6 + 8 = 14

14 = 14

Must click thanks and mark brainliest

What is the answer to this question i really need it asap, thank you.

Answers

Answer:

sorry I could not help you but I wish you luck

Which expressions are equivalent to 2 (5x - 3/4)? click two or more answers

A) -2 (5x) + (-2) (-3/4)
B) -10x - 3/4
C) -10x + 6/2
D) -10x + 3/2
E) -10x - 6/2

Answers

none of them actually. the answer would be 10x - 3/2

please help me is for my homework​

Answers

33.3 (repeating) hope this helps

Answer:

33.33%

Step-by-step explanation:

do divide 3÷9 and that is .333333333

and to convert a decimal to a percent you move the dot two number back so you get 33.33%

PLEASE HELP 25 MINUTES LEFT
38
Marlene wrote an arithmetic sequence where al
=
8 and the common difference is -3.
What is a rule for the nth term of this sequence?

Answers

Answer:

D

Step-by-step explanation:

The nth term of an arithmetic sequence is

[tex]a_{n}[/tex] = a₁ + (n - 1)d

where a₁ is the first term and d the common difference

Here a₁ = 8 and d = - 3 , then

[tex]a_{n}[/tex] = 8 - 3(n - 1) = 8 - 3n + 3 = - 3n + 11 → D

Show ALL STEPS You may use the Balance Strategy Game to help.

x/2+x/4=5

Answers

ohhhh, what? !?? balance ?

How do I solve part c

Answers

Answer:

A possible answer could be 2√2. It depends where points B and C would be. I guess this information was given in the parts A and B.

Step-by-step explanation:

See picture attached

Please help explanation if possible

Answers

Answer:

Step-by-step explanation:

Subtract 3x from both sides

3x - 3x - 2y = -3x + y

Combine

-2y = - 3x + 7  

Divide by - 2

-2y/-2 = -3x/-2 + 7/-2

y = 3x/2 - 7/2

(PICTURE) Im really struggling with questions like these at the moment, if you could please help me out thank you

Answers

Answer:

Arthur should select the Mount Joy Pool when there are less than 10 people at the party

Step-by-step explanation:

The initial fee to rent the Woodbridge Pool = $50

The additional fee per person after renting = $5

The initial fee to rent at Mount Joy Pool = 0 (no initial fee)

The fee per person at Mount Joy Pool = $10

The equation of a straight line is y = m·x + c

Ley y  represent the total cost of renting the pool, x, represent the number of persons in the pool, m represent the fee per person and c represent the initial charges, we get;

The total cost of renting Woodbridge Pool, y = 5·x + 50

The total cost of renting the Joy Pool, y = 10·x

Equating both values of y gives;

5·x + 50 = 10·x

∴ 5·x = 50

x = 50/5 = 10

x = 10

From the above equations, the cost of renting the pool is lower for the Joy Pool when there are less than 10 persons at the pool

It will cost the same amount to rent the pool when the number of persons in the pool are 10 persons and the cost of renting the Mount Joy Pool  will be more than the cost of renting the Woodbridge Pool, when there are more than 10 persons

Therefore, Arthur should select the Mount Joy Pool when there are less than 10 people at the party.

A bus has velocity 20m/s towards east and another bus has velocity 15m/s in West direction.If they start to move from a point simultaneously.What distances do they covered in 2 minutes.​

Answers

Answer:

4,200 m or 4.2 km.

First bus: 2400 m

2nd bus: 1800 m.

Step-by-step explanation:

Their combines speed is 20+15 = 35m/s.

Distance = speed * time

2 minutes = 120 seconds so

D = 35 * 120

= 4200 m.

Splitting the distances:

The first bus covers 20 * 120 = 2400m

The second bus covers 15*120 = 1800m.

Answer:

4200m

Step-by-step explanation:

If two objects A and B are moving in the opposite direction with speed xm/s and ym/s respectively,then;

Relative speed =(x+y)m/s

: Relative speed = 20+15=35m/s

time =2min=160sec

DISTANCE=SPEED ×TIME

D=35×120=4200

4200m

Which inequality is represented by the graph?

Answers

Answer:

It will be -5≤×≤4

I NEED HELP WITH THIS QUESTION ASAP ITS REALLY REALLY URGENT!!!!


Two kids, Albert and Bhara, are 20.0 m apart. Albert sees a soccer ball 25.0 m away. If the angle between the line formed by Albert and Bhara and the line from Albert to the soccer ball is 25 degrees, how far is Bhara from the soccer ball? Correctly round your answer to the nearest tenth of a meter.

Answers

Step-by-step explanation:

I can give you a formula SOH-Sinø=Opposite\Hypotenuse

CAH-Cos=Adjacent\Hypotenuse

TOA-Tan=Opposite\Adjacent

hi, please solve these three questions for me, i have to shoe solving steps.​

Answers

question 3

Step-by-step explanation:

i only able to show you the step of question 3..so sorry

From an 80ft building the angle of elevation of the top of a taller building is 59 degrees and the angle of depression of the base of this building is 65 degrees determine the height of the taller building

Answers

Answer:

First we find the distance between the two buildings =

Step-by-step explanation:

[tex] \frac{80}{ \tan(65) } = 37.3 ft[/tex]

then the upper part of the taller building =

[tex]37.3 \times \tan(59) = 62 \: ft[/tex]

Now the total height of the taller building

[tex]80 + 62 = 142 \: ft[/tex]

help please I'm doing some math homework ​

Answers

Answer:

Slope is 5/2

Step-by-step explanation:

[tex]{ \boxed{ \bf{slope = \frac{y _{2} - y _{1} }{x _{2} - x _{1}} }}}[/tex]

Substitute the terms:

[tex]{ \tt{slope = \frac{7 - ( - 3)}{4 - 0} }} \\ = \frac{5}{2} [/tex]

Five consecutive multiples of 7 have a sum of 350. What is the smallest of these numbers?
A. 70
B. 56
C. 77
D. 84

Answers

Answer:

B. 56

Step-by-step explanation:

x + (x + 7 ) + ( x + 14 ) + ( x + 21 ) + ( x + 28 ) = 350

( x + x + x + x + x ) + ( 7 + 14 + 21 + 28 ) = 350

5x + 70 = 350

     - 70    - 70

_____________

5x = 280

x = 56

Hope this helps!

The smallest number is 56, the correct option is B.

What is Sum?

The sum is the output of the mathematical operation, Addition.

Let the first number is x, they are multiples of 7,

As they are multiples of 7, the consecutive numbers will be added by 7 for next term.

The 5 consecutive numbers can be written as x, (x+7), (x+14), (x+21), (x+28).

The equation can be formed for the numbers that are given as,

An equation is a mathematical statement that relates an algebraic expression with other expression by an equal sign.

The sum of the multiples is 350

x + (x + 7 ) + ( x + 14 ) + ( x + 21 ) + ( x + 28 ) = 350

Grouping the variables and the constants separately

( x + x + x + x + x ) + ( 7 + 14 + 21 + 28 ) = 350

5x + 70 = 350

Adding (-70) to both sides of the equation

5x = 280

Dividing both sides by 5

x = 56

The value of the first number of the series is obtained.

The first value is the smallest number of the series.

To know more about Sum

https://brainly.com/question/13013054

#SPJ2

Write the equation in slope-intercept form. y = 6(x + 2) + 5x

Answers

Answer:

y=11x+12

Step-by-step explanation:

y = 6(x + 2) + 5x

y=6x+12+5x

y=11x+12

in slope interception form=  y=mx+c

                                                y=11x+12

Y=11x + 12 my brother

Will mark brainlest helppppppp​

Answers

Answer:

6

Step-by-step explanation:

again ?

7 = (3x - 4)/2

14 = 3x - 4

18 = 3x

x = 6

A train travels a distance of 60 km at uniform speed. If the speed of the train was reduced by 10 kmh-1, the time taken to travel the 60km will increase by 1/2h. Find the speed of the train at the beginning.

Answers

Answer:

Initial speed is 32 m/s

At uniform speed, acceleration is 0, (a = 0).

When speed reduced, (v - u) = 2.78 ms-¹, t = 1800 sec, s = 60 ,000 metres.

From first equation of motion:

[tex]{ \boxed{ \bf{v = u + at}}} \\ { \tt{(v - u) = at}}[/tex]

substitute:

[tex]{ \tt{2.78 = (a \times 1800)}} \\ { \tt{acceleration = 0.0015 \: {ms}^{ - 2} }}[/tex]

from second equation of motion:

[tex]{ \boxed{ \bf{s = ut + \frac{1}{2} a {t}^{2} }}}[/tex]

substitute:

[tex]{ \tt{60000 = 1800u + ( \frac{1}{2} \times 0.0015 \times {1800}^{2}) }} \\ { \tt{1800u = 57570}} \\ { \tt{u = 32 \: m {s}^{ - 1} }}[/tex]

Math models helpppp plss if you know about math models answer this pls

Answers

Answer:

A mathematical model is a description of a system using mathematical concepts and language. The process of developing a mathematical model is termed mathematical modeling. Mathematical models are used in the natural sciences and engineering disciplines, as well as in non-physical systems such as the social sciences.

Jeanette ice cream shop sold 10 sundaes with nuts and 7 sundaes without nuts to the total number of sundaes

Answers

The answer to the question is 17
There are 10 sundaes with nuts, and 7 without nuts. Our task is to find the total number of sundaes. We can find this by adding 10 + 7. This gives us 17 sundaes.

I suck at math ik but i need anyones help please

Answers

Answer:

f(2)= 1

f-¹(1)= 2

f-¹(f(2))= 2

I hope I helped you^_^

help me with the picture please

Answers

Answer:  115

=====================================================

Explanation:

Refer to the diagram below. I've drawn diagonal that slopes upward. This diagonal cuts the quadrilateral into two triangles: One is equilateral and the other is isosceles.

The equilateral triangle marked in blue has all three angles 60 degrees each.

Note that the 60 and y angles combined to form 130, so,

60+y = 130

y = 130-60

y = 70

Then focus on the isosceles triangle (angles y, w and w). These three interior angles must add to 180

y+w+w = 180

70+2w = 180

2w = 180-70

2w = 110

w = 110/2

w = 55

This adds onto its adjacent neighbor of 60 to get w+60 = 55+60 = 115 degrees which is the value of x.

A small bird can flap its wings 120 times in 30 s. What is the rate of change of wing flaps?

Answers

Answer:

4 Flaps per second

Step-by-step explanation:

M = 120/30

Answer:

4 times per second

Step-by-step explanation:

120 ÷ 30 = 4

30 ÷ 30 = 1

4 flaps per 1 second

so basically 4

To wash a window that is 4 meters off the ground, Rafi leans a 5-meter ladder against the side of the building. To reach the window, how far away from the building should Rafi place the base of the ladder?

Answers

Answer:

Base of the ladder is 3 meters away from the building.

Step-by-step explanation:

Let's use Pythagoras theorem to solve.

Pythagoras theorem says,

[tex]a^{2} +b^{2} =c^{2}[/tex]

Here let horizontal distance is "a''

Vertical distance of window is 4 m

So, b=4

The Rafi leans 5 m ladder against the wall. So, c=5.

[tex]a^{2} +4^{2} =5^{2}[/tex]

Simplify it

[tex]a^{2} +16=25[/tex]

Subtract both sides 16

[tex]a^{2} =9[/tex]

Take square root on both sides

a=±3

So, base of the ladder is 3 meters away from the building.

Nina is making pink candles to use as decorations on Valentine's Day. She melts red and white wax together and pours them into a heart-shaped mould. Then, she melts double the amount of red wax and double the amount of white wax together and pours them into a flower-shaped mould. Which candle is a lighter shade of pink?

Answers

Answer:

answer is neither, they are both the same shade.

Step-by-step explanation:

if she doubled the colors exactly then it wouldn't be any different. Also IXL told me I'm right :)

Answer:

Neither.

Step-by-step explanation:

Originally, she used the same amount. For the second one, she used double the same amount, but it was still the amount of red wax as of white wax. So, they both had the same amount, which means it is neither.

Please help !!!!!!!!!!!

Answers

Answer:

98°+62°+6x = 180

160+6x = 180

6x = 180 - 160

6x = 20

x= 20/6

x= 3.33

therefor your answer is 3.33

if line y bisects line AC, AB = 4 - 5x, and BC = 2x+25, find AC

Answers

Answer:

4-5x = 2x+25

-5x+4 = 2x+25

-7x = 21

x = -3

4 - 5(-3)

4 + 15

19

AC = 2 x 19

AC = 38

Let me know if this helps!

Find the missing value in each figure below. What does “y” equal?

Answers

Answer:

Step-by-step explanation:

The perpendicular is equal to 6. That's because the left triangle's missing angle is 180 - 45 -90 = 45

The angle in the right triangle is given as 52.

The cos(52) = adjacent side (which we just found to be 6) / y

Multiply both sides by y

y cos(52) = 6

cos(52) = 0.6157

Divide by sides by cos(52)

y = 6 / cos(52)

y = 6 / 0.6157

y = 9.76

Other Questions
who are you I don't know who were you The Olympic Games have been going on since last Monday. What's the negative sentence of it? TO BE ANSERED ASAPIf n = 4, then 98 9 n is equal to__________________. Which civilization depicted in the map remains largely a mystery to historians due to its yetuntranslatable written language?ArabianBar ofBrampalO Indus River Valley CivilizationO Yangtze River Valley CivilizationO Nile Valley CivilizationO Mesopotamian Civilization Ley de Charles-.1) En un recipiente hermtico se tiene 150 ml de una sustancia gaseosa, a una temperatura de 115C, esto como resultado de una reaccin. Cul sera su volumen inicial, cuando su temperatura era de 100C?2) Al inicio cuando la temperatura es de 200C, se tienen 350 ml de un gas Y. Qu volumen se obtendr si la temperatura se incrementa a 250C? [tex]( - 12) \times ( - 4) + ( - 8) + ( + 3)[/tex]could you help.. What is it that comes down, but does not go up?answer with intelligence. Hy cho bit gi tr v ngha ca s lng t n, l, m, ms khi m t trng thi ca electron trong nguyn t? A runner sprinted for 414 feet. How many yards is this? (2 + 6i)(8 9i)Divide and simp your school recently organized a work experience week , when each student in your class spent the week working in a local business of their choice. write an email to your friend about work experience week . 1. Samuel paid #34.20 for a blanket. If the marked price of the blanket is #41.78. What is the discount?2. A mother buys a dress for her daughter at a discount of 18%. If the price of the dress is #35.00. How does she actually pay for the dress? What is the distance between the following points?Will give brainliest What is cre in full eh? The business college computing center wants to determine the proportion of business students who have personal computers (PC's) at home. If the proportion exceeds 30%, then the lab will scale back a proposed enlargement of its facilities. Suppose 300 business students were randomly sampled and 65 have PC's at home. What assumptions are necessary for this test to be satisfied find the answer for 10 points for any Integer 'a',a 0 is _______give me answer (Ight ima need some help!! ) solve the following equation (2x + 3) (3x + 2) (3x + 2) (2x 3) = 0. solve for x and show work(2x + 3) (3x + 2) (3x + 2) (2x 3) = 0 Which of the eee-values satisfy the following inequality? Suppose Joe contracts with Marvin to frame out a shop at Joe's ranch for $10,000. During the framing Marvin discovers the costs of framing will be $2,500 more than he anticipated, and threatens to walk off the job unless Joe agrees to pay the additional $2,500. Joe agrees to the price increase. Most courts today would hold the agreement to pay the additional $2,500 to be _____ the agreement based on a(n) _______. (Choose two correct answers)